LSAT and Law School Admissions Forum

Get expert LSAT preparation and law school admissions advice from PowerScore Test Preparation.

 Administrator
PowerScore Staff
  • PowerScore Staff
  • Posts: 8919
  • Joined: Feb 02, 2011
|
#23418
Complete Question Explanation

Parallel Flaw—CE. The correct answer choice is (D)

The flaw in the reasoning found in this stimulus is a fairly common logical flaw—the notion that failure to prove something means that we can no longer make such a claim. In truth there are many theories which have not been definitively proved but are still generally accepted. The author presents this basic argument:

We cannot prove a link between the triceratops' extinction and asteroid impact, so we cannot claim that asteroid impact was the cause.

The correct answer choice will be guilty of the same logical fallacy: the mistaken notion that a failure to prove something definitively takes away our right to assert the claim.

Answer choice (A): The reasoning here is flawed, but this choice does not parallel the flawed logic found in the stimulus. In this case, the flaw is the mistaken belief that if two people aren't each individually capable of lifting something alone, they are not capable of lifting it as a pair either. This has nothing to do with a failure to definitively prove an assertion, so this answer choice is incorrect.

Answer choice (B): This answer choice presents a restatement that appears to be valid, so this cannot parallel the flawed logic found in the stimulus.

Answer choice (C): This answer choice is probably, for many, the most attractive of the wrong answer choices. It starts along the right lines, pointing out that two assertions have not been conclusively proven. The conclusion here, however, is that the opposite has been conclusively proven. This is subtly distinguishable from the flawed logic found in the stimulus, that the failure to prove an assertion means that we can no longer make that claim (not that the assertion is conclusively proven untrue).

The distinction here comes down to the strength of the conclusion: "we can't claim an assertion," versus that assertion is definitively false."

Answer choice (D): This is the correct answer choice, as it reflects the same flawed logic found in the stimulus: Fire officials could not definitively prove the link between the flood and the fire, so no such causal claim can be asserted. This is the same sort of flawed argumentation found in the stimulus and is therefore the correct answer.

Answer choice (E): This answer choice draws a conclusion that is much different from that drawn by the author of the stimulus. Here, based on reasonable beliefs, the author asserts that those beliefs are enough for us to narrow down the possible causes of the flood.
 jessicamorehead
  • Posts: 84
  • Joined: Jul 07, 2017
|
#44315
Can someone verify my line of reasoning?

I eliminated A, B and E based on their conclusions.
A: "I doubt that..." is not even close to "we CANNOT attribute..." in the stimulus' conclusion.

B: "either we know that Cheng will win or we know that Lin will win." I wasn't too crazy about the "either/or" presented in the conclusion, as well as it saying "WE KNOW"

C: this was temping, so I kept it as a contender. I ended up knocking it out because the stimulus had a causal relationship stated in it's premises, "The asteroid that hit the YP CAUSED both long term climatic change and a firestorm." The stimulus then went on to address these outcomes individually. However, this answer choice did not have an initial variable that caused the two things (John's driving at excessive speeds and John's weaving of lanes). Upon closer inspection, I see that this answer choice does have causal reasoning in it, but it's only presented in the conclusion. Is my logic okay for knocking this guy out?

D: Liked and kept as a contender. I found it to be correct because there was a variable (the flooding) which lead/caused two things (damage to furnace and a short in the electrical system). The answer then goes on to individually address the two things, saying they were not proven to cause the fire. The conclusion says we cannot say the original cause (the flood) caused the fire.

E: I didn't like how this one said "We have good reason to believe..." because the premises in the stimulus are more absolute and definitive. I also found this one was not logically flawed.

Let me know if my line of reasoning is okay, or if I'm reaching too far. Thanks!
 Daniel Stern
PowerScore Staff
  • PowerScore Staff
  • Posts: 81
  • Joined: Feb 07, 2018
|
#44354
For the most part, I'd say your logic is sound. Both B and E are logically valid. I'd be careful about some of your linguistic parsing: for instance, in analyzing answer choice C, you noted that the answer choice did not include "an initial causing variable," and I don't know that every parallel flaw credited response will track the stimulus that closely. If you saw the same logical flaw in C that was present in the stimulus, the fact that there was no initial cause element wouldn't make the answer wrong.

Overall, I'd say you are on the right track, because you're getting to the right answer.

Definitely be certain you are analyzing the stimulus and identifying the flaw in the logic before you move to the answer choices and try to puzzle out the language of each.

Good luck in your studies,
Dan
 cinnamonpeeler
  • Posts: 21
  • Joined: Apr 27, 2020
|
#75411
I'm not really convinced by the original explanation for why (C) is wrong. The original conclusion is that we cannot attribute T's extinction to the asteroid's impact. Is this not the same thing as saying "the asteroid did not cause T's extinction"? Isn't being unable to attribute an effect to a cause is the same as saying that the cause did not produce the effect?

I am also wondering if, in analyzing secondary flaws, it might be reasonable to think that if the asteroid had other effects, such as a tornado, it could be that these effects led to the extinction of Dino T. Or is it the case that when the speaker states that "the asteroid caused x and y," x and y can be the only effects of the cause?

Alternatively, could it be that the asteroid's impact itself killed the dinosaurs? In this case it is is possible for the premises to be true (i.e., that it was not the firestorm nor the climate change that caused the extinction) and the conclusion to be conclusion to be false (i.e., the asteroid did cause the extinction of the dinosaurs through its physical impact).

Is the reasoning in (E) also valid? It says that S or M ⇉ UHT ⇉ Fl. Thus, S or M ⇉ Fl. This seems to be to be a valid inference.
 Christen Hammock
PowerScore Staff
  • PowerScore Staff
  • Posts: 61
  • Joined: May 14, 2020
|
#75764
Hi Cinnamon!

You hit on the right distinction here, but there is a difference between not being able to conclusively attribute an effect to a cause and saying that the cause didn't product the effect. The former says that we don't have the evidence. Answer Choice (C) is definitely flawed because it conflates these two things.

The stimulus is using a different kind of flawed reasoning, though, and you already have it figured out! Just because two individual effects of the asteroid didn't make the triceratops go extinct doesn't mean that the asteroid did not cause the extinction. Answer Choice (D) most closely parallels this flaw. Just because two impacts of the flood can't be shown to have caused the fire, doesn't mean that we can rule out the flood altogether!

You're also correct about Answer Choice (E). If you have "good reason" to believe the elements of the logical chain (high tides leading to flooding; sun or moon causing high tides), it's "reasonable" to conclude that the sun or moon caused the flooding. This is valid reasoning.
 hope
  • Posts: 84
  • Joined: Feb 13, 2018
|
#77019
Could this be regarded as an error in the use of evidence? Lack of evidence for a position is taken to prove the position false?
 Adam Tyson
PowerScore Staff
  • PowerScore Staff
  • Posts: 5153
  • Joined: Apr 14, 2011
|
#77546
That's essentially what our explanation is saying, hope. Just because we cannot prove which of those two things caused it does not mean we cannot prove that it was one of them. That lack of proof isn't proof of anything, really. But it's not a really pure form of that flaw, because the author didn't go so far as to say "and therefore the asteroid impact didn't cause the extinction." THAT would be a more classic lack of evidence flaw - we cannot prove it is true, so it MUST be false. Here, it's just "we cannot prove it was caused by one of these two effects, so we cannot prove it was caused by any other related effect." Not that it's false, but that we cannot prove it.

I think another way to look at this one is as a part to whole flaw - just because we cannot prove that is was due to one of those two parts, the author thinks we cannot prove it was due to the whole. Maybe we can prove that it had to be one or the other of those two causes, or a combination of both, even if we cannot prove that it was specifically caused by one of them?

This is a good example of how Flaws often combine and overlap, and it's not all black and white. A great lesson for our LSAT studies overall!

Get the most out of your LSAT Prep Plus subscription.

Analyze and track your performance with our Testing and Analytics Package.